dayme11
Thanks Received: 0
Forum Guests
 
Posts: 8
Joined: June 23rd, 2010
 
 
 

Q13 - A recent study reveals that

by dayme11 Tue Sep 21, 2010 6:26 pm

I was b/w answer A & D. But chose D instead and don't understand why that answer is incorrect.
 
giladedelman
Thanks Received: 833
LSAT Geek
 
Posts: 619
Joined: April 04th, 2010
 
 
 

Re: Q13 - A recent study reveals that

by giladedelman Wed Sep 22, 2010 6:20 pm

Thanks for using the forum. The title of your post refers to PrepTest 34, but you've posted it in the PrepTest 38 section. Which test did you mean to refer to?
 
cdjmarmon
Thanks Received: 0
Elle Woods
Elle Woods
 
Posts: 59
Joined: July 12th, 2011
 
 
 

Re: Q13 -

by cdjmarmon Fri Apr 13, 2012 1:30 pm

I chose B over A. I see both B and A as good weaken answers but I felt B did a better job. However, when I read the Kaplan and Princeton review explanations B is considered "Out of scope."

I can see A is definitely a good weakener. BUt I dont see how B is so wrong its irrelevant, especially when answers to weaken questions can bring in new information.
 
timmydoeslsat
Thanks Received: 887
Atticus Finch
Atticus Finch
 
Posts: 1136
Joined: June 20th, 2011
 
This post thanked 2 times.
 
trophy
Most Thanked
trophy
First Responder
 

Re: Q13 -

by timmydoeslsat Sat Apr 14, 2012 11:36 pm

I would agree that a simple "out of scope" explanation is not very helpful. I certainly saw it as relevant.

So this argument concludes that TV advertising does not significantly affect children's preferences for breakfast cereals.

The evidence for this is that:

Group 1 (no tv) and Group 2 (tv) both preferred the sugary cereals being advertised on tv.

So what this argument is really saying is, "Hey look, that group 2 would have chosen the sugary cereal anyway. Those kids in group 1 picked that kind of cereal. So the ads are not affecting anything. The tv group is going after the same cereal as the no-tv kids."

I will give my spin on this one:

Choice C, for example, would absolutely weaken the argument had it stated the other group in this answer choice. The group that was not watching tv. Had this group been exposed to this commercial prior to the study, this would weaken the idea that tv ads are not affecting preferences.

A) This answer choice is telling us that the no-tv kids are being influenced by the tv kids. So this is challenging the idea of tv ads not being an influence on preference. The no-tv kids are essentially following what the tv kids are doing. And the tv kids may have been influenced by those ads.

B) This answer choice is not affecting how the argument concluded the no preference idea. It was due to the no-tv kids having the same preference as the tv kids. And since that was true, how is tv having an effect on their preference? As of now, it does seem possible that the tv kids would have chosen the sugary cereals had they not watched those commercials anyway, just look at the no-tv group, they did it.

This answer choice is strengthening the idea that the tv ads influenced the tv kid group. However, it is not challenging the conclusion that the tv ads are not significantly affecting the preferences for kids with cereal. While it may be true that the tv kids were influenced by these ads, that does not mean that this is significantly affecting their preferences. This is due to the fact that the kids who were not watching tv have the same preferences as the group that watch tv.

The arguer would still be able to reach their conclusion with ease.

He/she would simply say..."Yeah, group 1 was influenced to the sugar cereal, but the no-tv kids were preferring the same cereal. So those tv ads were not significantly affecting the preference."

We have to attack some assumptions made about the no-tv kids.

That is, that their preference was not affected by something else.
 
giladedelman
Thanks Received: 833
LSAT Geek
 
Posts: 619
Joined: April 04th, 2010
 
 
 

Re: Q13 -

by giladedelman Tue Apr 17, 2012 5:32 am

Yes, good explanation! I agree, "out of scope" really needs to be used sparingly; it definitely doesn't apply to answer (B)!

The trouble with (B) is that it merely says that, in general, it's possible for television to influence "the preference for sweets," but the point of the study is way more specific: when it comes to TV commercials for sugary cereals and kids, there's no significant influence going on. We need something to weaken the conclusion of this specific study. I do think (B) weakens it a teeny tiny bit, but (A) is a far superior answer--and remember, we are looking for the answer that most weakens.

By way of analogy, imagine if I argued that the weather in France is pretty nice. It wouldn't really weaken that argument if you pointed out that in some places, the weather is pretty lousy. I mean, yeah, okay, but what about France?
 
bp0
Thanks Received: 0
Forum Guests
 
Posts: 14
Joined: November 24th, 2013
 
 
 

Re: Q13 - A recent study reveals that

by bp0 Sun Mar 02, 2014 11:26 am

My issue here is this: The book states A is the correct answer. So A basically states that the kids who did not watch the advertising are simply influenced by the kids who did; which, those kids aren’t influenced, according to the paragraph, by advertising. So you got one group no T.V., one group watched T.V. Both groups supported the sugary cereal heavily advertised. How does that weaken the original argument. To me it just states that the kid who did not watch the advertising is influenced by the person who did; which, according to the argument isn’t influenced by the T.V. in the first place? So the other kid is just copying him the kid who watched T.V. who likes whatever he likes for some other reason other than advertising. How does that weaken the study that shows advertising does not affect children’s preferences? B) Directly states that the preference for treats can be influenced by environmental factors such as Television advertising. IF that is true, doesn’t that EXACTLY weaken the ENTIRE argument of the study: that advertising does not significantly affect children's preferences for breakfast cereals?
 
coco.wu1993
Thanks Received: 1
Elle Woods
Elle Woods
 
Posts: 64
Joined: January 06th, 2014
 
 
 

Re: Q13 - A recent study reveals that

by coco.wu1993 Fri Apr 18, 2014 5:20 am

I did this question right, but I'm not sure why C is wrong. If the children wathed these advertisements before, their identical preferences may attribute to the effects of advertisements which weakens the argument.
User avatar
 
maryadkins
Thanks Received: 641
Atticus Finch
Atticus Finch
 
Posts: 1261
Joined: March 23rd, 2011
 
This post thanked 1 time.
 
 

Re: Q13 - A recent study reveals that

by maryadkins Wed Apr 23, 2014 11:29 am

(C) doesn't weaken it for a couple of reasons. One is that it doesn't address the kids who didn't watch TV. If those kids still prefer the cereals, then the argument is pretty good"”that's the part we want to attack! The other reason is that if it's true that the kids who watched TV already knew about the cereals, maybe the TV didn't influence them...which would mean there's an argument that C slightly strengthens the argument!

Remember, we want to WEAKEN the claim that TV doesn't influence kids; we want to make it so that TV does influence kids.

As for the others that haven't yet been discussed in this thread:

(D) strengthens the argument; remember, we want them to be influenced by what they watch on TV, not immune to it.

(E) adults?

Hope this helps.
User avatar
 
WaltGrace1983
Thanks Received: 207
Atticus Finch
Atticus Finch
 
Posts: 837
Joined: March 30th, 2013
 
 
trophy
Most Thanked
trophy
Most Thankful
trophy
First Responder
 

Re: Q13 - A recent study reveals that

by WaltGrace1983 Sun May 11, 2014 1:44 pm

So if (C) said, "Most of the children in the group that had not watched television were already familiar with the advertisements for these cereals," then that would be a good weakener right? It casts doubt on the idea that TV advertising does not significantly affect children's preferences because it shows that they were already, in some way, affected by the TV ads to begin with. Just because you watched something during one specific study doesn't mean that you didn't watch the ads before and could therefore still be affected by them!
User avatar
 
maryadkins
Thanks Received: 641
Atticus Finch
Atticus Finch
 
Posts: 1261
Joined: March 23rd, 2011
 
This post thanked 1 time.
 
 

Re: Q13 - A recent study reveals that

by maryadkins Sat May 17, 2014 1:28 pm

Yes, I like that better"”then it starts to look more like (A). The children who had not watched TV were still being influenced by the ads.
 
bp0
Thanks Received: 0
Forum Guests
 
Posts: 14
Joined: November 24th, 2013
 
 
 

Re: Q13 - A recent study reveals that

by bp0 Thu May 29, 2014 7:11 am

What about B, anyone want to address what I wrote above?
User avatar
 
WaltGrace1983
Thanks Received: 207
Atticus Finch
Atticus Finch
 
Posts: 837
Joined: March 30th, 2013
 
 
trophy
Most Thanked
trophy
Most Thankful
trophy
First Responder
 

Re: Q13 - A recent study reveals that

by WaltGrace1983 Thu May 29, 2014 12:12 pm

bp0 Wrote:What about B, anyone want to address what I wrote above?


There a lot of problems with (B).

First of all, this doesn't hurt the link between the premise and the conclusion. The premise is about a study, the conclusion is about the implications of that study. While not ALL weaken/strengthen questions will always speak directly to the gap between the premises and the conclusion, this one is alarmingly irrelevant to the premises. In addition, understand that the conclusion is based on the study, "a recent study reveals...". We probably want to attack that study.

Secondly, we are talking SPECIFICALLY about CHILDREN. "Humans" goes way too far. So it is not a universal trait in humans but is it a universal trait in children? adults? infants? All we know is that not ALL humans have a preference for sweets. But so what? We want to know specifically about children.

Thirdly - and this is a very weak reason that bolsters the argument for why (B) is wrong but isn't necessarily sufficient for eliminating answer choices all the time - "can" is a very weak word. It can certainly be an element of a correct answer, but because (B) is already a little dicey it makes me a little hesitant.

In addition, (A) is simply much more relevant, much stronger, and much better than (B).
 
roflcoptersoisoi
Thanks Received: 0
Atticus Finch
Atticus Finch
 
Posts: 165
Joined: April 30th, 2015
 
 
 

Re: Q13 - A recent study reveals that

by roflcoptersoisoi Mon Jun 06, 2016 8:39 pm

First of all, I didn't like this question, particularly because the answer choice (A) requires the presumption that the children that watched TV were indeed influenced by the advertisements. This looked good early on but I was a bit apprehensive about selecting it, in the end I did after I eliminating the other answer choices using PoE.

(A) Looks good, keep for now. This suggests that tv advertisements affected the preferences of the children who did not watch TV indirectly, by influencing the children that did who in turn influenced them.
(B) This states that it is possible for other factors such as tv advertisements to influence preferences for sweets. Ok. cool, to what extent do they if they do? This doesn't weaken the contention that fact that advertisements don’t significantly affect ones preference for them.
(C) Just because they were familiar with the commercials doesn’t mean they were affected or influenced by them. Even if they were it wouldn't affect the argument. Eliminate
(D) This strengthens, eliminate.
(E) Irrelevant, we’re talking about children’s preferences

(A) is clearly the answer, pick it and move on.